M119 In-class Review Answers


\hsize=6truein
\newcount\qno
\def\qn {\par\advance\qno by1\noindent\strut
	\hbox to\parindent{\the\qno.\hfil}\hangindent\parindent\ignorespaces}
\font\bg=cmbx10 scaled \magstep2
\font\hg=cmr7 scaled 2985
\vglue 0pt
\rm
\qn $\displaystyle {3\over 2\sqrt x}-{2\over x^3}$

\qn $\displaystyle {2(x^2+9)-(2x+5)(2x)\over (x^2+9)^2}$

\qn %$h'(x)=5(6x^3-27)^4(18x^2)$ and 
    $h''(x)=20(6x^3-27)^3(18x^2)^2+5(6x^3-27)^4(36x)$

\qn $\displaystyle f'(x)=\lim_{h\to 0}{f(x+h)-f(x)\over h}$

\qn $29/7$

\qn $1/4$

\qn $-5$

\qn $\displaystyle x-1={1\over 2}(y-1)$

\qn $f'(x)=4x^3-12x^2=4x^2(x-3)$ so increasing
when $x>3$ and decreasing when $x<3$.
$f''(x)=12x^2-24x=12x(x-2)$ so
concave up when $x>2$ or $x<0$ and
concave down when $0<x<2$.

\qn $f(x)$ is discontinuous at $x=-1$ and $x=1$.

\qn $\displaystyle f'(x)={2(x^2+4)-4x^2\over (x^2+4)^2}=
			{-2(x-2)(x+2)\over (x^2+4)^2}=0$
implies $x=2$ or $-2$.  Check the endpoints $-2$ and $3$ as well.
$f(2)=1/2$, $f(-2)=-1/2$, $f(3)=6/13$.  Hence
the maximum is $1/2$ and the minimum is $-1/2$.

\qn
A graph of f'(x) is here.

\qn 
$P(x)=xp-C(x)=x(200-3x)-75-80x+x^2=-2x^2+120x-75$ so that
$P'(x)=-4x+120=0$ implies $x=30$.
Check the endpoints as well at $0$ and $40$.
$P(0)=-75$, $P(30)=1725$, $P(40)=1525$.
Hence the maximum profit occurs at $x=30$
with a price of $p=110$.

\qn 
$\displaystyle\lim_{h\to 0} {f(4+h)-f(4)\over h}=
	f'(4)={1\over 2\sqrt 4}=1/4$.

\qn $(f\circ g)'(7)=f'\big(g(7)\big)g'(7)
		=f'(3)4=8$